3
$\begingroup$

Suppose $ \mathbb{P} := (P, {\leq_P}, 1_P) $ is a separative partial order. Let $ \mathbb{B} := \operatorname{RO}(\mathbb{P}) $ denote the Boolean completion.

Fix some dense embedding $ i \colon P \to B^+ $. Then $ i $ is an order-isomorphism (one-to-one, order-preserving) to its image.

Assume that $ \mathbb{P} $ is isomorphic to each of its cones $ \mathbb{P} \mathord{\upharpoonright} p := \{ q \in P : q \leq_P p \} $ (where $ p \in P $ is some condition).

Question. Is $ \mathbb{B} $ isomorphic to each of its cones? If not, is $ \mathbb{B} $ isomorphic to each cone of the form $ \mathbb{B} \mathord{\upharpoonright} i(p) $ (where $ p \in P $) at least?

Attempt to proof the second statement. Fix $ p \in P $. Let $ f \colon \mathbb{P} \to \mathbb{P} \mathord{\upharpoonright} p $ denote an isomorphism. Then $ i \circ f \circ i^{-1} \colon \mathbb{B} \to \mathbb{B} \mathord{\upharpoonright} i(p) $ is a canonical candidate for the isomorphism since $ i \mathord{\upharpoonright} (\mathbb{P} \mathord{\upharpoonright} p) $ should be a dense embedding of $ \mathbb{P} \mathord{\upharpoonright} p $ into $ \mathbb{B} \mathord{\upharpoonright} i(p) $, right?

$\endgroup$

1 Answer 1

4
$\begingroup$

Your second statement is correct, simply because boolean completions are unique up to isomorphism.

For the stronger statement, let $b \in \mathbb{B}^+$. Let $\{ p_\alpha : \alpha < \kappa \}$ be a maximal antichain of elements of $i[\mathbb{P}]$ below $b$. Pick some maximal antichain $\{ q_\alpha : \alpha < \delta \}$ in $\mathbb{P}$ where $\delta \geq \kappa$. If $\delta > \kappa$, then using the fact that $\mathbb{P} \cong \mathbb{P} \restriction p_0$, enlarge the first antichain to one of size $\delta$; denote it $\{ p'_\alpha : \alpha < \delta \}$. So $\mathbb{B} \restriction b$ is isomorphic to the complete boolean sum of $\delta$ copies of $\mathbb{B}$, since $\mathbb{B} \cong \mathbb{B} \restriction p'_\alpha$ for each $\alpha$. But so is $\mathbb{B}$ itself.

$\endgroup$
4
  • $\begingroup$ Thanks for you answer! I have a couple of questions: (1) For the first part, you use $ \operatorname{RO}(\mathbb{P} \restriction p) \cong \operatorname{RO}(\mathbb{P}) \restriction i(p) $, right? Is this clear? What is about my attempt (as stated in the question)? (2) There's a typo: It should be $ \mathbb{P} \cong \mathbb{P} \restriction i^{-1}(p_0) $, right? So we find that antichain $ \{ p'_\alpha : \alpha < \delta \} $ of size $ \delta $ in $ (\mathbb{B} \restriction b) \cap i[\mathbb{P}] $. That's ok for me. $\endgroup$
    – Justus87
    Jul 23, 2014 at 10:43
  • $\begingroup$ (3) Is $ \{ p'_\alpha : \alpha < \delta \} $ a maximal antichain in $ \mathbb{B} \restriction b $? (4) Now, it seems like you use a theorem (which is unknown to me) that states: Given a maximal antichain in a complete Boolean algebra, this complete Boolean algebra is isomorphic to the complete Boolean sum of all the cones below the members of that antichain. Can you give a reference or is this clear? $\endgroup$
    – Justus87
    Jul 23, 2014 at 10:44
  • $\begingroup$ (5) If I understand your argument correctly, you apply the mentioned theorem twice: (A) to $ \mathbb{B} \restriction b $ and $ \{ p'_\alpha : \alpha < \delta \} $; (B) to $ \mathbb{B} $ and $ \{ i(q_\alpha) : \alpha < \delta \} $. Is this correct? Thanks in advance! $\endgroup$
    – Justus87
    Jul 23, 2014 at 10:45
  • $\begingroup$ (1) Yes, $i : \mathbb{P} \restriction p \to RO(\mathbb{P}) \restriction i(p)$ is dense, so the isomorphism follows. (2) Sure but both are correct. (3) Yes. (4) Yes it's clear, define the isomorphism in the obvious way. (5) Sure I guess. You're being extremely formal! $\endgroup$ Jul 23, 2014 at 15:24

Your Answer

By clicking “Post Your Answer”, you agree to our terms of service and acknowledge you have read our privacy policy.

Not the answer you're looking for? Browse other questions tagged or ask your own question.